scholarly journals The Lower Bounds of Eight and Fourth Blocking Sets and Existence of Minimal Blocking Sets

2007 ◽  
Vol 19 (3) ◽  
pp. 99-111
Author(s):  
L.Yasin Nada Yassen Kasm Yahya ◽  
Abdul Khalik
2011 ◽  
Vol 19 (4) ◽  
pp. 313-316 ◽  
Author(s):  
Antonio Cossidente ◽  
Oliver H. King

2003 ◽  
Vol 11 (3) ◽  
pp. 162-169 ◽  
Author(s):  
J. Barát ◽  
S. Innamorati

10.37236/5717 ◽  
2016 ◽  
Vol 23 (2) ◽  
Author(s):  
Jan De Beule ◽  
Tamás Héger ◽  
Tamás Szőnyi ◽  
Geertrui Van de Voorde

In this paper, by using properties of Baer subplanes, we describe the construction of a minimal blocking set in the Hall plane of order $q^2$ of size $q^2+2q+2$ admitting $1-$, $2-$, $3-$, $4-$, $(q+1)-$ and $(q+2)-$secants. As a corollary, we obtain the existence of a minimal blocking set of a non-Desarguesian affine plane of order $q^2$ of size at most $4q^2/3+5q/3$, which is considerably smaller than $2q^2-1$, the Jamison bound for the size of a minimal blocking set in an affine Desarguesian plane of order $q^2$.We also consider particular André planes of order $q$, where $q$ is a power of the prime $p$, and give a construction of a small minimal blocking set which admits a secant line not meeting the blocking set in $1$ mod $p$ points. Furthermore, we elaborate on the connection of this problem with the study of value sets of certain polynomials and with the construction of small double blocking sets in Desarguesian projective planes; in both topics we provide some new results.


10.37236/446 ◽  
2010 ◽  
Vol 17 (1) ◽  
Author(s):  
G. Van De Voorde

A small minimal $k$-blocking set $B$ in $\mathrm{PG}(n,q)$, $q=p^t$, $p$ prime, is a set of less than $3(q^k+1)/2$ points in $\mathrm{PG}(n,q)$, such that every $(n-k)$-dimensional space contains at least one point of $B$ and such that no proper subset of $B$ satisfies this property. The linearity conjecture states that all small minimal $k$-blocking sets in $\mathrm{PG}(n,q)$ are linear over a subfield $\mathbb{F}_{p^e}$ of $\mathbb{F}_q$. Apart from a few cases, this conjecture is still open. In this paper, we show that to prove the linearity conjecture for $k$-blocking sets in $\mathrm{PG}(n,p^t)$, with exponent $e$ and $p^e\geq 7$, it is sufficient to prove it for one value of $n$ that is at least $2k$. Furthermore, we show that the linearity of small minimal blocking sets in $\mathrm{PG}(2,q)$ implies the linearity of small minimal $k$-blocking sets in $\mathrm{PG}(n,p^t)$, with exponent $e$, with $p^e\geq t/e+11$.


2021 ◽  
Vol 72 ◽  
pp. 101814
Author(s):  
Nanami Bono ◽  
Tatsuya Maruta ◽  
Keisuke Shiromoto ◽  
Kohei Yamada

2015 ◽  
Vol 24 (1) ◽  
pp. 36-52
Author(s):  
Geertrui Van de Voorde

2008 ◽  
Vol 36 (2) ◽  
pp. 365-380 ◽  
Author(s):  
A. Abdollahi ◽  
M. J. Ataei ◽  
A. Mohammadi Hassanabadi

Sign in / Sign up

Export Citation Format

Share Document